MCQ Questions for Class 10 Maths Chapter 1 Real Numbers with Answers
Below you will find NCERT MCQ Questions for Class 10 Maths Chapter 1 Real Numbers with Answers which is very helpful during the preparation of examinations. These MCQ Online Test are one marks questions which will give you overview of the chapter in no time. One should try to understand Class 10 MCQ Questions as it is based on latest exam pattern released by CBSE.
Also, students can check NCERT Solutions for Class 10 Maths Chapter 1 for improving their marks and have good understanding of the chapter.

1. Two natural numbers whose sum is 85 and the least common multiple is 102 are:
(a) 30 and 55
(b) 17 and 68
(c) 35 and 55
(d) 51 and 34
Solution (d) 51 and 34
2. HCF of 8, 9, 25 is
(a) 8
(b) 9
(c) 25
(d) 1
Solution (d) 1
3. 7√3 is -
(a) An irrational
(b) A natural number
(c) A rational number
(d) None of these
Solution (a) An irrational
4. LCM of three numbers 28, 44, 132 is:
(a) 528
(b) 231
(c) 462
(d) 924
Solution (d) 924
5. The largest number which divides 615 and 963 leaving remainder 6 in each case is
(a) 82
(b) 95
(c) 87
(d) 93
Solution (c) 87
6. If A = 2n + 13, B = n + 7, where n is a natural number then HCF of A and B is:
(a) 2
(b) 1
(c) 3
(d) 4
Solution (b) 1
7. The largest number which divides 245 and 1029 leaving remainder 5 in each case is
(a) 4
(b) 8
(c) 12
(d) 16
Solution (d) 16
8. If 112 = q×6+r, then the possible values of r are:
(a) 2, 3, 5
(b) 0, 1, 2, 3, 4, 5
(c) 1, 2, 3, 4
(d) 0, 1, 2, 3
Solution (b) 0, 1, 2, 3, 4, 5
9. A rational number can be expressed as a non-terminating repeating decimal if the denominator has the factors
(a) Other than 2 or 5
(b) 2 or 3 only
(c) 2 or 5 only
(d) None of these
Solution (a) Other than 2 or 5
10. Which of the following numbers has terminating decimal expansion?
(a) 3/11
(b) 3/5
(c) 5/3
(d) 3/7
Solution (b) 3/5
11. The product of two numbers is -20/9. If one of the numbers is 4, find the other.
(a) –5/9
(b) 3/11
(c) 12/39
(d) –9/11
Solution (a) –5/9
12. If p is a positive prime integer, then √p is:
(a) A rational number
(b) An irrational number
(c) A positive integer
(d) None of these
Solution (b) An irrational number
13. The product of two consecutive integers is divisible by
(a) 2
(b) 3
(c) 5
(d) 7
Solution (a) 2
14. If HCF(a, b) = 12 and a × b = 1800, then LCM(a, b) is
(a) 150
(b) 90
(c) 900
(d) 1800
Solution (a) 150
15. Pairs of natural numbers whose least common multiple is 78 and the greatest common divisor is 13 are:
(a) 58 and 13 or 16 and 29
(b) 68 and 23 or 36 and 49
(c) 18 and 73 or 56 and 93
(d) 78 and 13 or 26 and 39
Solution (d) 78 and 13 or 26 and 39
16. If the HCF of 65 and 117 is expressible in the form 65m – 117, then the value of m is
(a) 4
(b) 2
(c) 1
(d) 3
Solution (b) 2
17. Find the greatest number of 5 digits, that will give us remainder of 5, when divided by 8 and 9 respectively.
(a) 99921
(b) 99931
(c) 99941
(d) 99951
Solution (c) 99941
18. The least positive integer divisible by 20 and 24 is
(a) 360
(b) 120
(c) 480
(d) 240
Solution (b) 120
19. Two natural numbers whose difference is 66 and the least common multiple is 360, are:
(a) 120 and 54
(b) 90 and 24
(c) 180 and 114
(d) 130 and 64
Solution (b) 90 and 24
20. The largest number which divides 70 and 125, leaving remainders 5 and 8 respectively, is
(a) 13
(b) 65
(c) 875
(d) 1750
Solution (a) 13
21. For any two positive integers a and b, there exist (unique) whole numbers q and r such that
(a) q = ar + b , 0 ⩽ r < b.
(b) a = bq + r , 0 ⩽ r < b.
(c) b = aq + r , 0 ⩽ r < b.
(d) None of these
Solution (b) a = bq + r , 0 ⩽ r < b.
22. The multiplicative inverse of zero is
(a) Is 1
(b) Is 0
(c) Is 1/0
(d) Does not exist
Solution (d) Does not exist
23. The largest number that will divide 398,436 and 542 leaving remainders 7,11 and 15 respectively is
(a) 17
(b) 11
(c) 34
(d) 45
Solution (a) 17
24. For some integer p, every even integer is of the form
(a) 2p + 1
(b) 2p
(c) p + 1
(d) p
Solution (b) 2p
Post a Comment